Course Content
All Previous Years Krok 2 Papers with Explanations
About Lesson

51. A 58 y.o. man complained of severe inspiratory dyspnea and expectoration of frothy and blood-tinged sputum. He has been suffering from essential hypertension and ischemic heart disease. On examination: acrocyanosis, “bubbling”breathing, Ps- 30/min, BP- 230/130 mm Hg, bilateral rales. Choose medicines for treatment.

A. Morphine, furosemide, nitroprusside sodium

B. Theophylline, prednisolon

C. Albuterol, atropine, papaverine

D. Strophanthine, potassium chloride, plathyphylline

E. Cordiamine, isoproterenol


Answer: Morphine, furosemide, nitroprusside sodium

Explanation

The patient’s symptoms and examination findings suggest a severe episode of acute pulmonary edema, which is a medical emergency that requires urgent treatment to improve respiratory function and reduce the risk of complications.   In this situation, the most appropriate treatment would be a combination of morphine, furosemide, and nitroprusside sodium.  

Morphine is a potent analgesic and sedative that can help relieve the patient’s dyspnea and anxiety by reducing pulmonary congestion and decreasing the work of breathing.   Furosemide is a loop diuretic that can help reduce fluid overload in the lungs and improve respiratory function by increasing the excretion of sodium and water by the kidneys.  

Nitroprusside sodium is a vasodilator that can help reduce blood pressure and improve cardiac function by dilating the blood vessels and decreasing the afterload on the heart.   Together, these medications can help reduce pulmonary congestion, improve respiratory function, and reduce the risk of complications in patients with acute pulmonary edema.  

Theophylline, prednisolone, albuterol, atropine, papaverine, strophanthine, potassium chloride, plathyphylline, cordiamine, and isoproterenol are not appropriate for the initial treatment of acute pulmonary edema and may even worsen the patient’s condition in some cases.


52. A patient has got a sudden attack of severe substernal pain at night. On examination: confusion, pallor of the skin, acrocyanosis, cold sweat, BP- 80/50 mm Hg, Ps- 120/min, irregular and weak pulse. What condition are these symptoms typical for?

A. Cardiogenic shock

B. Acute left-side heart failure

C. Acute right-side heart failure

D. Radicular syndrome

E. Acute vascular insufficiency


Answer: Cardiogenic shock 

Explanation

The symptoms described – severe substernal pain, confusion, pallor of the skin, acrocyanosis, cold sweat, low blood pressure, rapid and weak pulse – are typical of cardiogenic shock.  

Cardiogenic shock is a condition where the heart suddenly can’t pump enough blood to meet the body’s needs. It is most commonly caused by a heart attack, which can lead to damage to the heart muscle and impair its ability to pump blood effectively.  

The symptoms of cardiogenic shock include a sudden drop in blood pressure, rapid and weak pulse, confusion, cold and clammy skin, and shortness of breath.

Treatment for cardiogenic shock typically involves supportive measures such as oxygen, intravenous fluids, and medications to improve heart function, along with interventions to treat the underlying cause of the shock, such as revascularization procedures in cases of heart attack.


53. A 61 y.o. man complained of sneezing and substernal pain on exertion. In the last 2 weeks such pain appeared at rest, with increased frequency, and couldn’t be suppressed by 1 tablet of nitroglycerin. What is the most likely diagnosis?

A. Unstable angina pectoris

B. Angina pectoris of a new onset

C. Myocarditis

D. Radiculitis

E. Stable angina pectoris of the III functional class


Answer: Unstable angina pectoris 

Explanation

The symptoms described – substernal pain on exertion that has progressed to occur at rest, increased frequency, and unresponsive to nitroglycerin – are indicative of unstable angina pectoris.  

Unstable angina pectoris is a medical emergency and is caused by reduced blood flow to the heart due to partial or complete obstruction of the coronary arteries. It is characterized by chest pain or discomfort that occurs at rest, with increasing frequency, severity, and duration, and is not relieved by nitroglycerin or rest.  

In contrast, stable angina pectoris occurs during physical exertion or emotional stress and is relieved by rest or nitroglycerin. Angina pectoris of a new onset would also manifest as stable angina and is often related to an underlying cause like coronary artery disease.

Myocarditis is inflammation of the heart muscle and typically presents with symptoms like chest pain, fever, and fatigue. Radiculitis is inflammation of the nerve roots and typically presents with symptoms like shooting pain, numbness, and weakness.  

Therefore, given the patient’s symptoms, unstable angina pectoris is the most likely diagnosis, and immediate medical attention is required to prevent further complications like heart attack or sudden cardiac death.


54. A 41 y.o. woman complains of weakness, fatigue, fever up to 380C, rash on the face skin, pain in the wrists and the elbows. On physical examination: erythematous rash on the cheeks with “butterfly”look, the wrists and elbow joints are involved symmetrically, swollen, sensitive, friction rub over the lungs, the heart sounds are weak, regular, HR88/min, BP- 160/95 mm Hg. CBC shows anemia, leucopenia, lymphopenia; on urine analysis: proteinuria, leukocyturia, casts. What is the main mechanism of disease development?

A. Production of antibodies to doublestranded DNA

B. Production of myocytes antibodies

C. Production of antibodies to endothelial cells

D. Production of myosin antibodies

E. Production of antimitochondrial antibodies


Answer:  Production of antibodies to doublestranded DNA

Explanation

The patient’s symptoms and physical examination findings are suggestive of systemic lupus erythematosus (SLE), a chronic autoimmune disorder that can affect multiple organs and tissues.  

The “butterfly” rash on the face, joint involvement, and friction rub over the lungs are characteristic of SLE. The presence of anemia, leucopenia, lymphopenia, proteinuria, leukocyturia, and casts on urine analysis are also indicative of SLE-related kidney involvement.  

Antibodies to double-stranded DNA (anti-dsDNA) are a hallmark feature of SLE and are involved in the pathogenesis of the disease.

These antibodies bind to DNA and form immune complexes that deposit in various tissues, leading to inflammation and tissue damage.   In contrast, myocyte antibodies are associated with myocarditis, an inflammation of the heart muscle. Antibodies to endothelial cells are involved in the development of vasculitis, a condition characterized by inflammation and damage to blood vessels.

Myosin antibodies are associated with autoimmune myocarditis, while antimitochondrial antibodies are associated with primary biliary cirrhosis, a liver disorder.  

Therefore, based on the patient’s symptoms and laboratory findings, the main mechanism of disease development in this case is the production of antibodies to double-stranded DNA, which is characteristic of systemic lupus erythematosus.


55. A 56 y.o. woman has an acute onset of fever up to 390C with chills, cough, and pain on respiration in the right side of her chest. On physical examination: HR90/min, BP- 95/60 mm Hg, Ps- 26/min. There is dullness over the right lung. On X-ray: infiltrate in the right middle lobe of the lung en palpation. What is the diagnosis?

A. Community-acquired lobar pneumonia with moderate severity

B. Community-acquired bronchopneumonia

C. Acute pleurisy

D. Acute lung abscess

E. Hospital-acquired lobar pneumonia


Answer: Community-acquired lobar pneumonia with moderate severity

Explanation

The patient’s symptoms and physical examination findings suggest acute bacterial pneumonia. The acute onset of fever with chills, cough, and pain on respiration in the right side of the chest are typical of pneumonia.

The dullness over the right lung and infiltrate in the right middle lobe of the lung on X-ray support this diagnosis.   Community-acquired pneumonia (CAP) is an acute infection of the lungs that occurs outside of hospitals or other healthcare facilities. Lobar pneumonia is a type of CAP that affects a single lobe of the lung, and it is usually caused by Streptococcus pneumoniae.  

The severity of CAP is classified as mild, moderate, or severe based on the patient’s symptoms, physical examination findings, and laboratory results. The patient in this case has moderate severity pneumonia, as she has evidence of systemic inflammatory response with a high heart rate, low blood pressure, and tachypnea.  

In contrast, community-acquired bronchopneumonia is a diffuse form of pneumonia that affects multiple areas of the lung, and it is usually caused by a variety of bacterial pathogens. Acute pleurisy is an inflammation of the pleural membrane that covers the lungs and can cause chest pain.

An acute lung abscess is a collection of pus within the lung tissue, usually caused by an infection. Hospital-acquired pneumonia is an infection of the lungs that develops in patients who have been hospitalized for more than 48 hours and is usually caused by multidrug-resistant bacteria.  

Therefore, based on the patient’s symptoms, physical examination findings, and X-ray results, the most likely diagnosis is community-acquired lobar pneumonia with moderate severity.


56. A 29 y.o. woman is critically ill. The illness is presented by high fever, chills, sweating, aching pain in lumbar area, discomfort during urination and frequent voiding. Pasternatsky’s sign is positive in both sides. On lab examination: WBC20 ∗ 109/L; on urine analysis: protein – 0,6g/L, leukocyturia, bacteriuria. Your preliminary diagnosis.

A. Acute pyelonephritis

B. Exacerbation of chronic pyelonephritis

C. Acute glomerulonephritis

D. Acute cystitis

E. Nephrolithiasis


Answer: Acute pyelonephritis

Explanation

The patient’s symptoms and laboratory findings are consistent with acute pyelonephritis, which is an infection of the kidneys caused by bacteria that ascend from the urinary tract.  

The high fever, chills, sweating, aching pain in the lumbar area, discomfort during urination, frequent voiding, and positive Pasternatsky’s sign (flank pain upon percussion) are all typical symptoms of acute pyelonephritis.

The laboratory findings of leukocyturia and bacteriuria on urine analysis further support this diagnosis.   Exacerbation of chronic pyelonephritis may present with similar symptoms, but it usually occurs in patients with a history of recurrent urinary tract infections or underlying renal abnormalities.

Acute glomerulonephritis is an inflammation of the glomeruli (the filtering units of the kidneys) and typically presents with symptoms like hematuria, proteinuria, and edema. Acute cystitis is an infection of the bladder and can cause symptoms like dysuria, urinary urgency, and frequency.

Nephrolithiasis is the presence of stones in the kidneys or urinary tract and typically presents with symptoms like flank pain, hematuria, and urinary obstruction.  

Therefore, based on the patient’s symptoms and laboratory findings, the most likely diagnosis is acute pyelonephritis. Prompt treatment with antibiotics is required to prevent complications like sepsis, kidney abscess, or renal failure.


57. A 45 y.o. man has complained of having epigastric and right subcostal aching pain, pruritus, indigestion, dark color of the urine and acholic stool, fever and significant weight loss for 1 month. On examination: jaundice, presence of Curvuasier’s sign. US scan did not reveal stones in the gallbladder and choledochus. What is the most likely diagnosis?

A. Cancer of the pancreas head

B. Gallbladder stones

C. Chronic pancreatitis

D. Chronic cholangitis

E. Chronic hepatitis


Answer: Cancer of the pancreas head

Explanation

The patient’s symptoms and physical examination findings are suggestive of pancreatic cancer, specifically cancer of the pancreas head. The epigastric and right subcostal aching pain, pruritus, indigestion, dark urine, acholic stool, jaundice, and weight loss are all symptoms commonly associated with pancreatic cancer.  

Curvuasier’s sign, which is the presence of a palpable mass in the left upper quadrant due to an enlarged gallbladder caused by obstruction of the common bile duct, is also indicative of pancreatic cancer.  

The absence of gallbladder stones on ultrasound scan does not rule out pancreatic cancer as a cause of the patient’s symptoms, as pancreatic cancer can cause obstruction of the common bile duct and lead to jaundice and other symptoms.  

Chronic pancreatitis may cause similar symptoms, but it usually occurs in patients with a history of alcohol abuse or other underlying conditions.

Gallbladder stones and chronic cholangitis may cause similar symptoms, but the absence of stones on ultrasound scan and the presence of Curvuasier’s sign suggest an obstructive process within the pancreas or common bile duct. Chronic hepatitis may cause jaundice and weight loss, but it does not typically cause acholic stool or Curvuasier’s sign.  

Therefore, based on the patient’s symptoms and physical examination findings, the most likely diagnosis is cancer of the pancreas head. Further diagnostic testing, such as CT scan or MRI, may be required to confirm the diagnosis and determine the extent of the disease.


58. A 27 y.o. man complained of aching epigastric pain right after meal, heartburn and nausea. Stomach endoscopy revealed a large amount of mucus, hyperemia and edema of mucous membrane in gastric fundus with areas of atrophy. Make a diagnosis.

A. Chronic gastritis of type A

B. Chronic gastritis of type B

C. Peptic ulcer of stomach

D. Chronic gastritis of type C

E. Menetrier’s disease


Answer:  Chronic gastritis of type A

Explanation

The patient’s symptoms and endoscopic findings are indicative of chronic gastritis, a condition characterized by inflammation of the stomach lining. The aching epigastric pain right after meals, heartburn, and nausea are typical symptoms of chronic gastritis.  

The endoscopic findings of a large amount of mucus, hyperemia, and edema of the mucous membrane in the gastric fundus with areas of atrophy are consistent with chronic gastritis of type A, also known as autoimmune gastritis.

This type of gastritis is caused by an autoimmune reaction that leads to the destruction of the parietal cells in the stomach, which produce stomach acid and intrinsic factor. The destruction of these cells can lead to reduced acid secretion, impaired absorption of vitamin B12, and increased risk of gastric cancer.  

Chronic gastritis of type B, on the other hand, is caused by Helicobacter pylori infection and typically affects the antrum of the stomach. Peptic ulcer of the stomach is a complication of chronic gastritis and is characterized by a break in the mucous membrane of the stomach lining.

Chronic gastritis of type C is caused by chemical irritants, such as nonsteroidal anti-inflammatory drugs (NSAIDs) or alcohol. Menetrier’s disease is a rare condition characterized by hypertrophic gastropathy and protein-losing enteropathy.  

Therefore, based on the patient’s symptoms and endoscopic findings, the most likely diagnosis is chronic gastritis of type A, also known as autoimmune gastritis. Further testing, such as serology for autoantibodies or biopsy for histopathological examination, may be required to confirm the diagnosis.


59. A 25 y.o. woman complained of fatigue, hair loss and brittle nails. The examination revealed pallor of skin, Ps- 94/min, BP- 110/70 mm Hg. On blood count: Hb- 90 g/L, RBC- 3, 5 ∗ 1012/L, C.I.- 0,7; ESR- 20 mm/h. Serum iron level was 8,7 mcmol/L. What treatment would you initiate?

A. Ferrous sulfate orally

B. Iron dextrin injections

C. Vitamin B12 intramuscularly D. Blood transfusion

E. Packed RBCs transfusion


Answer: Ferrous sulfate orally

Explanation

The patient’s symptoms and laboratory findings are consistent with iron deficiency anemia, a condition in which there is a decrease in the number of red blood cells due to a lack of iron.

The fatigue, hair loss, brittle nails, pallor of skin, and low hemoglobin level are all typical symptoms of iron deficiency anemia.  

The serum iron level of 8.7 mcg/L is also consistent with iron deficiency anemia, as it is below the normal range.   The treatment for iron deficiency anemia is to replenish the body’s iron stores. Oral ferrous sulfate is the first-line treatment for mild to moderate iron deficiency anemia.

Iron dextran injections may be used in severe cases or when oral iron is not well tolerated. Vitamin B12 injections are used to treat pernicious anemia, a condition caused by a deficiency in intrinsic factor, which is needed for the absorption of vitamin B12.

Blood transfusion is reserved for severe cases of anemia when there is a risk of organ damage or life-threatening complications.   T

herefore, based on the patient’s symptoms and laboratory findings, the most appropriate treatment is ferrous sulfate orally to replenish the body’s iron stores and treat the iron deficiency anemia.

The patient should also be advised to eat a diet rich in iron, such as red meat, poultry, fish, beans, and leafy green vegetables, to help maintain adequate iron levels. Follow-up blood tests should be done to monitor the response to treatment.

60. A 60 y.o. woman has had increased BP up to 210/110 mm Hg for the last 7 years. On examination: heart apex is displaced to the left. There are signs of left ventricular hypertrophy on ECG. What is the most probable diagnosis?

A. Essential hypertension, 2nd stage

B. Essential hypertension, 1st stage

C. Symptomatic hypertension

D. Cardiomyopathy

E. Ischemic heart disease

Answer:  Essential hypertension, 2nd stage

Explanation

The patient’s history of increased blood pressure up to 210/110 mm Hg for the last 7 years, along with the signs of left ventricular hypertrophy on ECG, suggest a diagnosis of essential hypertension, specifically 2nd stage hypertension.  

Essential hypertension is a condition characterized by persistently elevated blood pressure without any identifiable cause. The American College of Cardiology/American Heart Association (ACC/AHA) guidelines classify hypertension based on blood pressure readings, with 2nd stage hypertension defined as a systolic blood pressure of 140 mm Hg or higher or a diastolic blood pressure of 90 mm Hg or higher.  

Left ventricular hypertrophy is a common complication of long-standing hypertension, which can lead to increased myocardial oxygen demand, impaired diastolic function, and increased risk of heart failure and sudden cardiac death.  

Symptomatic hypertension refers to cases where there are signs or symptoms of organ damage or dysfunction due to hypertension, such as hypertensive retinopathy, renal dysfunction, or cerebrovascular disease. In this case, there is no evidence of symptomatic hypertension, but the long-standing hypertension has led to left ventricular hypertrophy.  

Cardiomyopathy and ischemic heart disease are other conditions that can cause left ventricular hypertrophy, but the patient’s history of long-standing hypertension and absence of other risk factors for these conditions make essential hypertension a more likely diagnosis.  

Therefore, based on the patient’s history, examination findings, and ECG results, the most probable diagnosis is essential hypertension, 2nd stage, with left ventricular hypertrophy as a complication. Management should focus on blood pressure control and risk factor modification to prevent further organ damage and cardiovascular events.


61. A 30 y.o. man complains of intense pain, skin reddening in the region of ankle joint, temperature rise up to 390. He fell ill suddenly. In the past there were such onsets that lasted for 5-6 days and didn’t cause any residual changes of the joint. The skin over the joint is hyperemic, without distinct outlines and infiltrative bank at the periphery. What is the most probable diagnosis?

A. Gout

B. Infectional arthritis

C. Rheumatoid arthritis

D. Erysipelatous inflammation

E. Osteoarthrosis


Answer: Gout

Explanation

The patient’s symptoms of sudden onset of intense pain, skin reddening, and temperature rise in the region of the ankle joint are consistent with an acute attack of gout, a type of arthritis caused by the deposition of uric acid crystals in the joints.  

The patient’s history of previous onsets that lasted for 5-6 days without causing any residual changes of the joint is also suggestive of gout, as gout attacks typically resolve spontaneously within a few days and do not cause permanent joint damage unless left untreated.  

The hyperemic skin over the joint without distinct outlines and infiltrative bank at the periphery is also a characteristic feature of acute gout, known as erythema migrans.   Infectional arthritis may present with similar symptoms, but there is usually a history of recent infection or underlying immunodeficiency.

Rheumatoid arthritis typically presents with symmetrical joint involvement and may cause joint deformities over time. Erysipelatous inflammation is a bacterial skin infection characterized by a well-defined area of erythema, edema, and pain.

Osteoarthrosis is a degenerative joint disease that typically presents with joint pain, stiffness, and limitation of movement, but it does not cause sudden onset of symptoms or skin changes.  

Therefore, based on the patient’s symptoms and history, the most probable diagnosis is gout. Treatment may include nonsteroidal anti-inflammatory drugs (NSAIDs), colchicine, and/or corticosteroids to relieve pain and inflammation, as well as lifestyle modifications to reduce uric acid levels and prevent future gout attacks.


62. A 6 y.o. asthmatic child was taken to the emergency hospital because of severe coughing and wheezing for the last 24 hours. Physical examination reveals that the child is excitable, has intercostal and suprasternal retractions, expiratory wheezing throughout all lung fields, RR60/min. Initial treatment may include the prescription of:

A. Subcutaneous epinephrine

B. Parenteral phenobarbital

C. Intravenous fluids in the first 2 h to compensate water deficiency

D. N-acetyl cysteine and cromolyn by inhalation

E. Parenteral gentamicyn


Answer: Subcutaneous epinephrine

Explanation

The child’s symptoms of severe coughing, wheezing, intercostal and suprasternal retractions, expiratory wheezing throughout all lung fields, and a respiratory rate of 60 breaths per minute are consistent with a severe asthma exacerbation.  

Subcutaneous epinephrine, also known as adrenaline, is an effective treatment for severe asthma exacerbations. It works by relaxing the smooth muscles of the airways, reducing inflammation, and improving breathing. Epinephrine can be given via subcutaneous injection in an emergency setting to quickly relieve symptoms and prevent respiratory failure.  

Parenteral phenobarbital is not an appropriate treatment for asthma exacerbations and may cause sedation and respiratory depression.   Intravenous fluids may be given in severe cases of asthma exacerbation to maintain hydration and electrolyte balance, but it is not the initial treatment for acute asthma exacerbations.  

N-acetyl cysteine and cromolyn are medications that may be used for the prevention and management of asthma, but they are not effective in treating acute severe exacerbations.   Parenteral gentamicin is an antibiotic that may be used to treat bacterial infections, but it is not indicated for the treatment of asthma exacerbations.  

Therefore, based on the child’s symptoms, the most appropriate initial treatment for this severe asthma exacerbation is subcutaneous epinephrine to quickly relieve symptoms and prevent respiratory failure. The child should be closely monitored and may require additional treatments, such as inhaled bronchodilators and corticosteroids, to control inflammation and prevent future exacerbations.


63. A surgical department admitted a newborn boy with foamy discharges from nose and mouth, cyanosis attacks. X-ray examination: blind end of esophagus is at the level of the II thoracic vertebra, gastric air bubble is under the left cupula of diaphragm. What is the most probable diagnosis?

A. Esophagus atresia, tracheo-esophageal fistula

B. Total esophagus atresia

C. Paraesophageal hernia

D. Esophagus atresia without a fistula

E. Bronchoesophageal fistula


Answer: Esophagus atresia, tracheo-esophageal fistula

Explanation

The newborn’s symptoms of foamy discharges from nose and mouth, as well as cyanosis attacks, suggest an abnormal connection between the trachea and esophagus, known as a tracheo-esophageal fistula.

This condition is commonly associated with esophageal atresia, a congenital condition in which the esophagus is either completely or partially blocked.   The X-ray findings of a blind end of the esophagus at the level of the II thoracic vertebra and a gastric air bubble under the left cupula of diaphragm are consistent with esophageal atresia with a distal tracheo-esophageal fistula.

In this condition, the upper portion of the esophagus ends in a blind pouch, while the lower portion of the esophagus is connected to the trachea, allowing air to pass into the stomach.   Total esophageal atresia is a rare condition in which the entire esophagus is blocked, without a connection to the trachea.

Paraesophageal hernia is a condition in which a portion of the stomach protrudes through the diaphragm next to the esophagus. Esophageal atresia without a fistula is a type of esophageal atresia in which there is no connection between the esophagus and the trachea or stomach.

Bronchoesophageal fistula is an abnormal connection between the bronchus and the esophagus, which is a separate condition from tracheo-esophageal fistula.   Therefore, based on the newborn’s symptoms and X-ray findings, the most probable diagnosis is esophageal atresia with a distal tracheo-esophageal fistula.

Immediate surgical intervention is necessary to correct the anomaly and prevent complications, such as aspiration pneumonia or respiratory distress syndrome.


64. A 30 y.o. man complains of sharp pain in the right ear, hearing loss, high temperature for three days. Objectively: right ear whispering language – 0,5 m, external ear is intact, otoscopically – eardrum protrusion, hyperemia and swelling, loss of landmarks. What disease is it?

A. Acute purulent otitis media

B. Acute mastoiditis

C. Chronic secretory otitis media

D. Chronic purulent otitis media

E. Eustachian tube disfunction


Answer: Acute purulent otitis media

Explanation

The patient’s symptoms of sharp pain in the right ear, hearing loss, and high temperature for three days are consistent with acute otitis media, a condition characterized by inflammation and infection of the middle ear.  

The objective finding of a whispering language of 0.5 m in the right ear suggests a significant hearing loss, which can be caused by middle ear effusion and eardrum involvement.   The otoscopic findings of eardrum protrusion, hyperemia, and swelling, as well as loss of landmarks, are also consistent with acute otitis media.

The presence of pus or fluid behind the eardrum may indicate a purulent infection, which can cause severe pain and hearing loss.   Acute mastoiditis is a complication of acute otitis media, in which the infection spreads to the mastoid bone behind the ear. It can cause additional symptoms such as swelling, redness, and tenderness over the mastoid bone, as well as fever and headache.  

Chronic secretory otitis media is a long-standing condition characterized by the presence of fluid in the middle ear without signs of infection. It may cause hearing loss or recurrent ear infections.   Chronic purulent otitis media is a chronic infection of the middle ear with persistent ear discharge and hearing loss.  

Eustachian tube dysfunction is a condition in which the tube connecting the middle ear to the back of the throat fails to open and close properly. This can cause pressure changes in the middle ear, leading to symptoms such as hearing loss, ear fullness, and tinnitus.  

Therefore, based on the patient’s symptoms and otoscopic findings, the most probable diagnosis is acute purulent otitis media, which is usually treated with antibiotics and pain relievers.

The patient should be closely monitored for complications, such as mastoiditis or hearing loss, and may require drainage of the middle ear if the infection does not respond to treatment.


65. A youth, aged 15, from childhood suffers from atopic dermatitis and allergy to the shellfish. In the last 3 months after acquiring aquarium fish rhinitis, conjunctivitis, itching in the nose developed. Level of what immunologic index should be defined in this case?

A. IgE

B. IgJ

C. IgM

D. IgA

E. Circulating immunocomplexes


Answer: IgE

Explanation

The patient’s history of atopic dermatitis and allergy to shellfish suggests a predisposition to allergic reactions. The recent development of rhinitis, conjunctivitis, and itching in the nose after acquiring aquarium fish is consistent with an allergic reaction to fish or other allergens in the aquarium environment.  

IgE is an immunoglobulin that is involved in the allergic response. It is produced by plasma cells in response to allergen exposure and binds to specific receptors on mast cells and basophils, triggering the release of histamine and other inflammatory mediators.

Therefore, the level of IgE can be used as an indicator of allergic sensitization and may help identify the specific allergens responsible for the patient’s symptoms.   IgJ is an immunoglobulin that plays a role in the transport of other immunoglobulins across mucosal surfaces. IgM is an immunoglobulin that is involved in the primary immune response to infections.

IgA is an immunoglobulin that is found in secretions, such as saliva and breast milk, and provides protection against pathogens in mucosal surfaces. Circulating immunocomplexes are complexes of antigens and antibodies that can be detected in the blood in various autoimmune and infectious diseases.  

Therefore, in this case, the level of IgE should be defined to help diagnose the patient’s allergic reaction to aquarium fish or other allergens in the environment. Allergy testing, such as skin prick testing or specific IgE blood testing, may also be recommended to identify the specific allergens responsible for the patient’s symptoms and guide management strategies.


66. A pregnant woman (35 weeks), aged 25, was admitted to the hospital because of bloody discharges. In her medical history there were two artificial abortions. In a period of 28-32 weeks there was noted the onset of hemorrhage and USD showed a placental presentation. The uterus is in normotonus, the fetus position is transversal (Ist position). The heartbeats is clear, rhythmical, 140 bpm. What is the further tactics of the pregnant woman care?

A. To perform a delivery by means of Cesarean section

B. To perform the hemotransfusion and to prolong the pregnancy

C. To introduct the drugs to increase the blood coagulation and continue observation

D. Stimulate the delivery by intravenous introduction of oxytocin

E. To keep the intensity of hemorrhage under observation and after the bleeding is controlled to prolong the pregnancy


Answer: To perform a delivery by means of Cesarean section

Explanation

The pregnant woman’s symptoms of bloody discharges, along with her history of two previous artificial abortions and a placental presentation detected by ultrasound, suggest the presence of placenta previa, a condition in which the placenta partially or completely covers the cervix.  

Placenta previa is a serious pregnancy complication that can cause heavy bleeding, preterm delivery, and fetal distress. In this case, the patient is already at 35 weeks of gestation, which increases the risk of severe bleeding and fetal compromise.  

The transverse position of the fetus also makes vaginal delivery unsafe, as it increases the risk of fetal head entrapment and shoulder dystocia.   Therefore, the most appropriate tactic for the pregnant woman’s care is to perform a delivery by means of Cesarean section.

This will minimize the risk of bleeding and fetal distress, and ensure the safe delivery of the baby.   Hemotransfusion may be necessary if the patient experiences significant bleeding during or after the delivery. Drugs to increase blood coagulation may also be used to control bleeding, but they are not a definitive treatment for placenta previa.  

Stimulating delivery by intravenous introduction of oxytocin is not recommended in cases of placenta previa, as it can cause further bleeding and fetal distress.   Keeping the intensity of hemorrhage under observation and prolonging the pregnancy is not a safe option in this case, as the risk of severe bleeding and fetal compromise increases as the pregnancy progresses.  

Therefore, based on the patient’s symptoms and ultrasound findings, the most appropriate tactic for her care is to perform a delivery by means of Cesarean section to ensure the safe delivery of the baby and minimize the risk of complications.


67. Studying of pulmonary tuberculosis incidence provided data about patients’ socioeconomic living conditions and bad habits. What method allows to estimate the impact of these factors on tuberculosis incidence?

A. Calculation of correlation coefficient

B. Calculation of correspondence index

C. Calculation of regression coefficient

D. Standardized index calculation

E. Calculation of reliability coefficient


Answer: Calculation of correlation coefficient

Explanation

Correlation coefficient is a statistical measure that quantifies the strength and direction of the linear relationship between two variables. In this case, the variables of interest are the socioeconomic living conditions and bad habits of patients, and the incidence of pulmonary tuberculosis.  

By calculating the correlation coefficient between these variables, it is possible to estimate the degree to which these factors are associated with the incidence of tuberculosis. A positive correlation coefficient indicates that higher levels of socioeconomic disadvantage and bad habits are associated with higher incidence of tuberculosis, while a negative correlation coefficient indicates an inverse relationship.  

Calculation of correspondence index is a method used to compare the distribution of two variables across categories or groups. It can be used to identify differences or similarities in the distribution of variables, but it does not quantify the strength or direction of the relationship between them.  

Calculation of regression coefficient is a statistical method used to estimate the relationship between a dependent variable and one or more independent variables. It can be used to predict the value of the dependent variable based on the values of the independent variable(s).  

Standardized index calculation is a method used to compare the values of a variable to a reference value or standard. It can be used to identify outliers or extreme values in a data set.  

Calculation of reliability coefficient is a statistical measure used to assess the consistency and stability of a measurement over time or across different raters or methods. It is not directly related to the estimation of the impact of socioeconomic living conditions and bad habits on tuberculosis incidence.  

Therefore, in this case, the most appropriate method to estimate the impact of socioeconomic living conditions and bad habits on tuberculosis incidence is the calculation of correlation coefficient.


68. A 15 y.o. girl was examined. Her medical history registers gradual onset of fever, malaise, loss of weight. There was nothing typical about the kind of fever which has been present for more than 7-10 days and changed quickly. Physical examination didn’t give evident results. What is the only most important examination for excluding miliary tuberculosis?

A. Chest X-ray

B. Liver or bone marrow biopsy

C. Tuberculin skin testing

D. Sputum smear and culture of m. tuberculosis

E. Bronchoscopy


Answer:  Chest X-ray

Explanation

Miliary tuberculosis is a form of tuberculosis that occurs when Mycobacterium tuberculosis spreads to multiple organs through the bloodstream, causing the formation of tiny granulomas. It can present with non-specific symptoms such as fever, malaise, and weight loss, and can be difficult to diagnose without appropriate testing.  

A chest X-ray is the most important examination for excluding miliary tuberculosis, as it can reveal the presence of multiple small nodules throughout both lungs, which are typical radiographic findings in miliary tuberculosis. Other imaging modalities, such as computed tomography (CT) or magnetic resonance imaging (MRI), may also be used to confirm the diagnosis.  

Liver or bone marrow biopsy may be useful to confirm the diagnosis of miliary tuberculosis if the chest X-ray is inconclusive or if there is evidence of organ involvement. However, these procedures are invasive and carry a risk of complications, so they are not routinely performed unless necessary.  

Tuberculin skin testing is used to screen for latent tuberculosis infection, but it cannot diagnose active tuberculosis or miliary tuberculosis.   Sputum smear and culture of M. tuberculosis are important diagnostic tests for pulmonary tuberculosis, but they are not useful for diagnosing miliary tuberculosis, as the bacilli may not be present in the sputum.  

Bronchoscopy may be useful in evaluating patients with suspected pulmonary tuberculosis, but it is not the most important examination for excluding miliary tuberculosis.   Therefore, in this case, the most important examination for excluding miliary tuberculosis is a chest X-ray, which can reveal the presence of multiple small nodules throughout both lungs, a characteristic radiographic finding in miliary tuberculosis.


69. A patient has got pain in the axillary area, rise of temperature developed 10 hours ago. On examination: shaky gait is evident, the tongue is coated with white coating. The pulse is frequent. The painful lymphatic nodes are revealed in the axillary area. The skin is erythematous and glistering over the lymphatic nodes. What is the most probable diagnosis?

A. Bubonic plague

B. Acute purulent lymphadenitis

C. Lymphogranulomatosis

D. Anthrax E. Tularemia


Answer: Bubonic plague

Explanation

The patient’s symptoms of pain in the axillary area, rise of temperature, shaky gait, coated tongue, and frequent pulse are consistent with the systemic symptoms of bubonic plague, a highly contagious bacterial infection caused by the bacterium Yersinia pestis.  

The painful lymphatic nodes in the axillary area, accompanied by erythematous and glistering skin over the lymph nodes, are characteristic of bubonic plague, which is the most common form of the disease. Other symptoms may include headache, chills, muscle aches, and fatigue.  

Acute purulent lymphadenitis is a localized infection of the lymph nodes, usually caused by bacteria, and typically presents with symptoms such as pain, redness, and swelling in the affected area. However, it does not cause systemic symptoms such as fever, shaky gait, or coated tongue.  

Lymphogranulomatosis, also known as Hodgkin’s lymphoma, is a type of cancer that affects the lymphatic system. It typically presents with symptoms such as painless swelling of the lymph nodes, fever, night sweats, and weight loss. It does not cause symptoms such as erythematous and glistering skin over the lymph nodes, or a coated tongue.  

Anthrax is a bacterial infection caused by Bacillus anthracis, which typically presents with symptoms such as skin lesions, fever, and malaise. It does not cause symptoms such as painful lymphatic nodes, erythematous and glistering skin over the lymph nodes, or a coated tongue.  

Tularemia is a bacterial infection caused by Francisella tularensis, which typically presents with symptoms such as fever, chills, headache, and muscle aches. It may also cause skin ulcers or lymph node swelling. However, it does not cause symptoms such as erythematous and glistering skin over the lymph nodes, or a coated tongue.  

Therefore, based on the patient’s symptoms and physical examination findings, the most probable diagnosis is bubonic plague. The patient should be isolated and treated with antibiotics as soon as possible to prevent further spread of the infection.


70. A 17 y.o. patient complains of acute pain in the knee joint and t 0- 380C. He was ill with angina 3 weeks ago. Objectively: deformation and swelling of the knee joints with skin hyperemia. Small movement causes an acute pain in the joints. Which diagnose is the most correct?

A. Rheumatism, polyarthritis

B. Systemic lupus erythematodes

C. Reactive polyarthritis

D. Infectious-allergic polyarthritis

E. Rheumatoid arthritis


Answer: Rheumatism, polyarthritis

Explanation

The patient’s symptoms of acute pain in the knee joint, fever, and skin hyperemia, along with a recent history of angina, suggest a rheumatic condition. Rheumatism, or rheumatic fever, is an inflammatory disease that can affect the joints, heart, skin, and other organs, and is caused by an abnormal immune response to a previous bacterial infection, such as streptococcal pharyngitis.

Polyarthritis is a type of arthritis that affects multiple joints, and is a common manifestation of rheumatic fever. The deformation and swelling of the knee joints, along with acute pain on movement, are typical of polyarthritis.   Systemic lupus erythematosus (SLE) is an autoimmune disease that can affect multiple organs, including the joints, skin, and kidneys.

It may present with symptoms such as joint pain, fever, and skin rash, but it is less likely in this case due to the recent history of angina.   Reactive polyarthritis is a type of arthritis that occurs as a reaction to an infection in another part of the body, such as the gastrointestinal tract or genitourinary system.

However, the recent history of angina suggests a rheumatic condition rather than a reactive arthritis.   Infectious-allergic polyarthritis is a type of arthritis that occurs as a result of an allergic reaction to an infectious agent, such as a virus or bacteria.

However, the recent history of angina suggests a rheumatic condition rather than an infectious-allergic arthritis.   Rheumatoid arthritis is a chronic autoimmune disease that primarily affects the joints, causing inflammation, pain, and joint damage.

It typically presents in older adults and is less likely in a 17-year-old patient with a recent history of angina.   Therefore, based on the patient’s symptoms and history, the most correct diagnosis is rheumatism, polyarthritis, which is a common manifestation of rheumatic fever. The patient should be treated with anti-inflammatory medications and antibiotics, and monitored for any cardiac complications.

71. A girl is 12 y.o. Yesterday she was overcooled. Now she is complaining on pain in suprapubic area, frequent painful urination by small portions, temperature is 37, 80C. Pasternatsky symptom is negative. Urine analysis: protein – 0,033 g/L, WBC- 20-25 in f/vis, RBC- 1-2 in f/vis. What diagnosis is the most probable?

A. Acute cystitis

B. Dysmetabolic nephropathy

C. Acute glomerulonephritis

D. Acute pyelonephritis

E. Urolithiasis


Answer: Acute cystitis

Explanation

The patient’s symptoms of pain in the suprapubic area, frequent painful urination, and low-grade fever are typical of acute cystitis, which is a common bacterial infection of the bladder that can often occur after overcooling. The negative Pasternatsky sign, which indicates the absence of pain in the kidneys when percussed, suggests that the infection is limited to the bladder.  

The urine analysis results show the presence of leukocytes and a small amount of red blood cells, which are indicative of inflammation in the bladder. The absence of significant proteinuria suggests that the infection has not affected the kidney function.  

Dysmetabolic nephropathy is a type of kidney disease that is caused by metabolic disorders, such as diabetes or hypertension, and typically presents with symptoms such as proteinuria, hypertension, and decreased kidney function. It is less likely in this case due to the absence of significant proteinuria and Pasternatsky sign.  

Acute glomerulonephritis is an immune-mediated disease that affects the glomeruli of the kidneys, and typically presents with symptoms such as hematuria, proteinuria, and hypertension. It is less likely in this case due to the absence of significant proteinuria and the negative Pasternatsky sign.  

Acute pyelonephritis is a bacterial infection of the kidneys, and typically presents with symptoms such as fever, flank pain, and nausea. The absence of significant proteinuria and the negative Pasternatsky sign suggest that the infection is limited to the bladder rather than involving the kidneys.  

Urolithiasis, or kidney stones, typically presents with symptoms such as severe flank pain, nausea, and vomiting, and may cause hematuria or urinary tract infection. It is less likely in this case due to the absence of significant flank pain and the presence of symptoms suggestive of bladder infection.  

Therefore, based on the patient’s symptoms and urine analysis results, the most probable diagnosis is acute cystitis, which is a common bacterial infection of the bladder that can often occur after overcooling. The patient should be treated with antibiotics and advised to maintain good hygiene and hydration to prevent further infections.

72. A 56 y.o. patient has worked at the aluminium plant over 20 years. Within 3 last years he has got loosening of teeth, bone and joint pains, piercing pains in heart area, vomiting. The preliminary diagnosis is:

A. Fluorine intoxication

B. Mercury intoxication

C. Lead intoxication

D. Phosphorus intoxication

E. Manganese intoxication


Answer: Fluorine intoxication

Explanation

The patient’s symptoms of loosening of teeth, bone and joint pains, piercing pains in the heart area, and vomiting are all consistent with the effects of chronic exposure to excessive amounts of fluoride, a toxic element that is commonly found in industrial processes such as aluminum production.  

Fluoride intoxication can cause a variety of symptoms and health problems, including dental and skeletal fluorosis, which can lead to brittle bones, joint pain, and tooth decay. Fluoride can also affect the cardiovascular system, causing symptoms such as chest pain, palpitations, and arrhythmias. Vomiting and gastrointestinal symptoms are also common.  

Mercury, lead, phosphorus, and manganese intoxications can also cause a variety of symptoms depending on the level and duration of exposure. However, the patient’s symptoms, along with his occupation in the aluminum industry, suggest that fluoride intoxication is the most likely cause.  

Mercury intoxication can cause a range of symptoms, including neurological problems, mood changes, and gastrointestinal symptoms. However, it is less likely in this case due to the absence of neurological symptoms and the patient’s occupation in the aluminum industry.  

Lead intoxication can cause symptoms such as abdominal pain, anemia, and neurological problems. However, it is less likely in this case due to the absence of anemia and the patient’s occupation in the aluminum industry.   Phosphorus intoxication can cause symptoms such as abdominal pain, muscle weakness, and respiratory distress.

However, it is less likely in this case due to the absence of respiratory symptoms and the patient’s occupation in the aluminum industry.   Manganese intoxication can cause symptoms such as tremors, rigidity, and cognitive impairment.

However, it is less likely in this case due to the absence of neurological symptoms and the patient’s occupation in the aluminum industry.  

Therefore, based on the patient’s symptoms and occupation in the aluminum industry, the most probable diagnosis is fluoride intoxication. The patient should be evaluated for his exposure and treated with measures such as discontinuing exposure, increasing calcium and vitamin D intake, and chelation therapy to remove fluoride from the body.


73. A 38 y.o. woman complains of a purulent discharge from the left nostril. The body temperature is 37, 50C. The patient has been ill for a week and associates her illness with common cold. There are a pain and tenderness on palpation of her left cheek. The mucous membrane in the left nasal cavity is red and turgescent. The purulent exudate is seen in the middle meatus in maxillary. What is the most probable diagnosis?

A. Acute purulent maxillary sinusitis

B. Acute purulent frontitis

C. Acute purulent ethmoiditis

D. Acute purulent sphenoiditis

E. –


Answer: Acute purulent maxillary sinusitis

Explanation

The patient’s symptoms of purulent discharge from the left nostril, pain and tenderness on palpation of the left cheek, and red and turgescent mucous membrane in the left nasal cavity are all consistent with acute maxillary sinusitis, which is a bacterial infection of the maxillary sinuses located in the cheeks.  

The presence of purulent exudate in the middle meatus of the maxillary sinus further supports the diagnosis of maxillary sinusitis.   Acute purulent frontitis, or frontal sinusitis, is a bacterial infection of the frontal sinuses located in the forehead, and typically presents with symptoms such as headache, fever, and nasal congestion.

The absence of significant frontal headache makes it less likely in this case.   Acute purulent ethmoiditis, or ethmoid sinusitis, is a bacterial infection of the ethmoid sinuses located between the eyes, and typically presents with symptoms such as nasal congestion, headache, and pain between the eyes.

The absence of significant pain between the eyes makes it less likely in this case.   Acute purulent sphenoiditis, or sphenoid sinusitis, is a bacterial infection of the sphenoid sinuses located behind the nose, and typically presents with symptoms such as headache, neck pain, and visual disturbances.

The absence of significant neck pain and visual disturbances makes it less likely in this case.   Therefore, based on the patient’s symptoms and the location of the purulent exudate, the most probable diagnosis is acute purulent maxillary sinusitis.

The patient should be treated with antibiotics and nasal decongestants, and advised to maintain good hydration and nasal hygiene to prevent further infections.


74. Condition of a parturient woman has been good for 2 hours after live birth: uterus is thick, globe-shaped, its bottom is at the level of umbilicus, bleeding is absent. The clamp put on the umbilical cord remains at the same level, when the woman takes a deep breath or she is being pressed over the symphysis with the verge of hand, the umbilical cord drows into the vagina. Bloody discharges from the sexual tracts are absent. What is the doctor’s further tactics?

A. To do manual removal of afterbirth

B. To apply Abduladze method

C. To apply Crede’s method

D. To do curettage of uterine cavity

E. To introduct oxitocine intravenously


Answer:  To do manual removal of afterbirth

Explanation

The description of the uterus as thick, globe-shaped, and with a bottom at the level of the umbilicus suggests that the placenta has not yet been expelled from the uterus, which is a potential complication after delivery.  

The fact that the umbilical cord is not rising with inspiration and that it is descending into the vagina with pressure on the abdomen suggests that the placenta is still attached to the uterine wall and has not yet separated.  

Given that the placenta has not been expelled within two hours after delivery, the most appropriate next step is to perform manual removal of the placenta to prevent the risk of bleeding and infection.  

Abduladze method and Crede’s method are techniques used to assist with the delivery of the placenta, but they are not appropriate in this case since the placenta has not yet separated.   Curettage of the uterine cavity may be necessary if there is retained placental tissue or excessive bleeding after delivery, but it is not the first-line treatment in this case.  

Oxytocin is a medication that can be used to help stimulate uterine contractions and aid in the expulsion of the placenta, but it is not appropriate in this case since the placenta has not yet separated.  

Therefore, based on the description of the patient’s condition, the most appropriate next step is to perform manual removal of the placenta to prevent the risk of bleeding and infection. The procedure should be performed by a trained healthcare professional in a sterile environment.

75. A 34 y.o. patient has been suffering from pulmonary tuberculosis for 7 years; he complains of muscle weakness, weight loss, diarrhea, frequent urination. Objectively: hyperpigmentation of skin, gums, internal surface of cheeks. AP- 90/58 mm Hg. Blood count: RBC- 3, 1 ∗ 1012/L, Hb- 95 g/L, C.I.- 0,92; leukocytes – 9, 4 ∗ 109/L, eosinophils – 7, segmentonuclear leukocytes – 45, stab neutrophils – 1, lymphocytes – 40, monocytes – 7, Na+- 115 mmol/L, +- 7,3 mmol/L. What is the preliminary diagnosis?

A. Primary adrenocortical insufficiency

B. Pheochromocytoma

C. Primary hyperaldosteronism

D. Congenital adrenocortical hyperplasia

E. Diabetes insipidus

Answer: Primary adrenocortical insufficiency

Explanation

The patient’s symptoms of muscle weakness, weight loss, and hyperpigmentation of the skin and mucous membranes are all consistent with Addison’s disease, which is a rare autoimmune disorder characterized by the progressive destruction of the adrenal cortex, leading to a deficiency of cortisol and aldosterone.  

The patient’s blood pressure is low (90/58 mm Hg), which can be a sign of adrenal insufficiency and a deficiency of aldosterone, which is necessary for maintaining blood pressure.   The patient’s blood count shows a low hemoglobin level (95 g/L), which can be a sign of anemia, a common complication of Addison’s disease.  

The patient’s electrolyte imbalance, with low sodium (115 mmol/L) and high potassium (7.3 mmol/L), is also consistent with adrenal insufficiency and a deficiency of aldosterone, which regulates sodium and potassium balance.   Pheochromocytoma, primary hyperaldosteronism, and congenital adrenocortical hyperplasia are all conditions that can affect the adrenal glands, but they typically present with different symptoms and laboratory findings.  

Diabetes insipidus is a condition that affects the kidneys and can cause excessive thirst and urination, but it is not typically associated with the symptoms and laboratory findings seen in this case.  

Therefore, based on the patient’s symptoms, laboratory findings, and electrolyte imbalance, the most probable diagnosis is primary adrenocortical insufficiency, or Addison’s disease. The patient should be managed with hormone replacement therapy, including glucocorticoids and mineralocorticoids, to replace the deficient hormones and manage the symptoms.


76. A 52 y.o. hard smoker complains of persistent cough with purulent sputum discharge especially in the mornings, dyspnea provoked even by slight physical exercises, wheezing chest, tachypnoe, general weakness. He considers himself to be ill for 12 years. The foresaid presentations appear 3-4 times per year usually after a common cold and have tendency to progress. What disease do you think about first of all?

A. Chronic obstructive lung disease

B. Bronchial asthma

C. Mucoviscidosis (cystic fibrosis)

D. Bronchoectatic disease

E. Aspergillosis


Answer: Chronic obstructive lung disease

Explanation

The patient’s symptoms of persistent cough with purulent sputum, dyspnea on exertion, wheezing, tachypnea, and general weakness are all consistent with COPD, which is a chronic respiratory disease typically caused by long-term exposure to irritants such as cigarette smoke.  

The fact that the symptoms tend to occur 3-4 times per year after a common cold and have a tendency to progress further supports the diagnosis of COPD, as exacerbations are common in this condition and can lead to a worsening of symptoms and lung function.  

Bronchial asthma can also cause symptoms such as coughing, wheezing, and dyspnea, but it typically presents with a reversible obstruction of the airways and is often triggered by allergens or exercise. Mucoviscidosis (cystic fibrosis) is a genetic disorder that affects the respiratory and digestive systems, and typically presents with symptoms such as chronic cough, recurrent lung infections, and digestive problems such as malabsorption.

However, the onset of symptoms in adulthood and the absence of digestive symptoms make it less likely in this case.   Bronchiectasis is a condition characterized by permanent dilation of the bronchi and is often associated with chronic respiratory infections. It can cause symptoms such as chronic cough, sputum production, and dyspnea, but it typically presents with a history of recurrent respiratory infections and is less likely to be associated with smoking.  

Aspergillosis is a fungal infection that can affect the lungs and typically presents with symptoms such as cough, dyspnea, and fever. However, it is less likely in this case given the absence of fever and the chronic nature of the symptoms.   Therefore, based on the patient’s symptoms, history of smoking, and tendency to have recurrent exacerbations, the most probable diagnosis is chronic obstructive lung disease (COPD).

The patient should be evaluated further with spirometry and imaging studies to assess lung function and rule out other potential causes of the symptoms. Treatment may include smoking cessation, bronchodilators, corticosteroids, and antibiotics as necessary.


77. 3 weeks ago a patient was ill with tonsillitis. Clinical examination reveals edema, arterial hypertension, hematuria, proteinuria (1,8 g/per day), granular and erythrocital casts. What is the preliminary diagnosis?

A. Glomerulonephritis

B. Cystitis

C. Pyelonephritis

D. Intestinal nephritis

E. Renal amyloidosis

Answer: Glomerulonephritis

Explanation

The most probable diagnosis in this case is A. Glomerulonephritis.   The patient’s symptoms of edema, arterial hypertension, hematuria, proteinuria, and granular and erythrocyte casts are all consistent with glomerulonephritis, which is an inflammatory disorder that affects the glomeruli of the kidneys and can lead to kidney damage and dysfunction.  

The fact that the patient had a recent history of tonsillitis suggests that the glomerulonephritis may be post-streptococcal in nature, which is a common cause of acute glomerulonephritis.   Cystitis and pyelonephritis are both infections of the urinary tract that can cause symptoms such as hematuria and proteinuria, but they typically present with symptoms such as dysuria, frequency, and urgency, which are not mentioned in this case.  

Intestinal nephritis is a rare complication of inflammatory bowel disease that can cause glomerulonephritis, but it is less likely in this case given the absence of gastrointestinal symptoms.   Renal amyloidosis is a condition in which abnormal proteins build up in the kidneys and can cause kidney damage and dysfunction.

It can cause symptoms such as proteinuria, edema, and hypertension, but it typically presents with a more gradual onset of symptoms and is less likely in this case given the recent history of tonsillitis.   Therefore, based on the patient’s symptoms and recent history of tonsillitis, the most probable diagnosis is glomerulonephritis.

The patient should be further evaluated with laboratory tests, including blood tests and a kidney biopsy, to confirm the diagnosis and assess the extent of kidney damage. Treatment may include supportive care, such as controlling blood pressure and managing fluid and electrolyte balance, as well as immunosuppressive therapy to reduce inflammation and prevent further kidney damage.


78. 47 y.o. patient complains of intensive skin itching, jaundice, bone pain. The skin is hyperpigmentated. There are multiple xanthelasma palpebrae. The liver is +6 cm enlarged, solid with acute edge. The blood analysis revealed total bilirubin – 160 mkmol/L, direct – 110 mkmol/L, AST2,1 mmol/L, ALT- 1,8 mmol/L, alkaline phosphotase – 4,6 mmol/L, cholesterol9,2 mmol/L, antimitochondrial antibodies M2 in a high titer. What is the probable diagnosis?

A. Primary biliary liver cirrhosis

B. Primary liver cancer

C. Chronic viral hepatitis

B D. Acute viral hepatitis B

E. Alcoholic liver cirrhosis


Answer:  Primary biliary liver cirrhosis

Explanation

The patient’s symptoms of skin itching, jaundice, bone pain, and hyperpigmentation of the skin, along with the presence of xanthelasma palpebrae, are all consistent with PBC, which is a chronic autoimmune disorder that primarily affects the bile ducts in the liver and can lead to liver damage and cirrhosis.  

The enlarged liver with a solid edge, along with the laboratory findings of elevated bilirubin, alkaline phosphatase, and cholesterol, further support the diagnosis of PBC, as these are all common findings in this condition.   The presence of antimitochondrial antibodies M2 in a high titer is also highly suggestive of PBC, as these antibodies are present in up to 95% of patients with this condition.  

Primary liver cancer is a possibility in patients with chronic liver disease, but it typically presents with different symptoms and laboratory findings, such as abdominal pain, weight loss, and elevated alpha-fetoprotein levels.   Chronic viral hepatitis and acute viral hepatitis B can cause liver damage and jaundice, but they typically present with different symptoms and laboratory findings, such as elevated liver enzymes and the presence of viral markers.  

Alcoholic liver cirrhosis is another possible cause of liver damage and cirrhosis, but it typically presents with a history of alcohol abuse and different laboratory findings, such as elevated gamma-glutamyl transferase levels.   Therefore, based on the patient’s symptoms, physical examination, laboratory findings, and the presence of antimitochondrial antibodies M2, the most probable diagnosis is primary biliary liver cirrhosis (PBC).

The patient should be further evaluated with imaging studies, such as ultrasound or MRI, to assess the extent of liver damage and cirrhosis. Treatment may include ursodeoxycholic acid to improve liver function and delay the progression of the disease, as well as other supportive measures such as vitamin supplementation and management of symptoms.


79. In the gynecologic office a 28 y.o. woman complains of sterility within three years. The menstrual function is not impaired. There were one artificial abortion and chronic salpingo-oophoritis in her case history. Oral contraceptives were not used. Her husband’s analysis of semen is without pathology. What diagnostic method will you start from the workup in this case of sterility?

A. Hysterosalpingography

B. Hormone investigation

C. Ultra sound investigation

D. Diagnostic scraping out of the uterine cavity

E. Hysteroscopia

Answer: Hysterosalpingography

Explanation

The patient’s history of chronic salpingo-oophoritis and one previous artificial abortion suggests that she may have tubal damage or blockage, which can be a common cause of infertility in women.   Hysterosalpingography is a radiologic procedure in which a contrast agent is injected into the uterus and fallopian tubes, followed by X-rays to visualize the anatomy and assess the patency of the tubes.

This test can help to identify any blockages or abnormalities in the fallopian tubes, which can be a cause of infertility.   Hormone investigations, such as measuring follicle-stimulating hormone (FSH), luteinizing hormone (LH), and estradiol levels, can also be useful in assessing ovarian function and identifying hormonal imbalances that may be contributing to infertility.  

Ultrasound investigation can help to identify any structural abnormalities in the uterus or ovaries, such as fibroids or ovarian cysts, but it may not be able to detect tubal blockages.   Diagnostic scraping out of the uterine cavity (endometrial biopsy) and hysteroscopy are both invasive procedures that may be useful in diagnosing certain conditions that can cause infertility, such as endometrial polyps or uterine adhesions, but they may not be able to identify tubal blockages.  

Therefore, based on the patient’s history and symptoms, the most appropriate diagnostic method to start from would be hysterosalpingography to assess the patency of the fallopian tubes and identify any tubal blockages or abnormalities. Depending on the results of this test, further investigations and treatments may be recommended to address any identified issues.


80. A patient has undergone an operation on account of perforated ulcer of stomach, terminal phase of diffuse peritonitis and endotoxic shock. In the postoperative period he is prescribed artificial pulmonary ventilation with 60% oxygen inhalation. Blood gases: 2- 70- 78 mm Hg, hypoxemy doesn’t ecrease, CVP (central venous pressure) – 150- 180 mm of water column, AP- 90/60 mm Hg (against the backgound of taking big doses of dopamine). Radiogram shows diffuse pulmonary infiltration. What cause the refractory arterial hypoxemia?

A. Respiratory distress syndrome

B. Bilateral pneumonia

C. Pneumothorax

D. Mendelson’s syndrome

E. Pulmonary edema


Answer: Respiratory distress syndrom

Explanation

The patient’s history of perforated ulcer of the stomach, diffuse peritonitis, and endotoxic shock suggests that he may have developed RDS as a complication of his critical illness and mechanical ventilation.   RDS is a condition characterized by acute lung injury and inflammation, which can lead to impaired gas exchange and refractory hypoxemia.

It can be caused by a variety of factors, including sepsis, trauma, and mechanical ventilation, which can lead to damage to the alveolar-capillary membrane and a loss of functional lung tissue.   The patient’s blood gas values of 2-70-78 mm Hg, along with the diffuse pulmonary infiltrates seen on the radiogram, are consistent with RDS.

The high CVP and low blood pressure, despite high doses of dopamine, suggest that the patient may also be experiencing cardiovascular compromise as a result of the RDS.   Bilateral pneumonia is another possible cause of diffuse pulmonary infiltrates and hypoxemia, but it is less likely in this case given the patient’s history of peritonitis and endotoxic shock, which suggest a systemic inflammatory response rather than a focal infection.  

Pneumothorax and pulmonary edema are also possible causes of hypoxemia, but they typically present with different clinical features and radiographic findings than RDS.   Mendelson’s syndrome is a rare complication of aspiration of gastric contents, which can lead to chemical pneumonitis and respiratory failure. However, it is less likely in this case given the patient’s history of perforated ulcer and diffuse peritonitis, which suggest a different mechanism of lung injury.  

Therefore, based on the patient’s clinical presentation, blood gas values, and radiographic findings, the most probable cause of refractory arterial hypoxemia in this case is respiratory distress syndrome (RDS) secondary to the patient’s underlying critical illness and mechanical ventilation.

Treatment may include supportive care, such as optimizing mechanical ventilation and providing supplemental oxygen, as well as addressing any underlying factors contributing to the RDS, such as sepsis or fluid overload.


81. According to results of medical and pedagogical observation during the gymnastics lesson in the 9th grade there was plotted a phisiological curve characterized by gradual increasing of pulse rate during the opening part of lesson, 80% increase during the main part; the curve has 4 waves. How can the lesson’s structure be assessed?

A. Long interval between exercises

B. The lesson’s structure is correct

C. Exercise stress is insufficient

D. Exercise stress is excesive

E. Exercise stress is adequate


Answer: Long interval between exercises

Explanation

The lesson’s structure can be assessed as having a long interval between exercises, as indicated by the physiological curve characterized by a gradual increase in pulse rate during the opening part of the lesson, a significant increase during the main part, and 4 waves.  

A physiological curve with a gradual increase in pulse rate during the opening part of the lesson is expected as the body prepares for physical activity. However, a significant increase in pulse rate during the main part of the lesson suggests that the exercise stress is too intense, and the body is struggling to keep up.

The presence of 4 waves in the curve indicates that the exercise stress is not well-tolerated by the body, and the body is experiencing fluctuations in response to the exercise.   Therefore, the lesson’s structure can be assessed as having a long interval between exercises, which is causing the body to experience periods of inactivity and then sudden bursts of intense physical activity.

This type of exercise pattern can be challenging for the body to handle, leading to fluctuations in pulse rate and other physiological responses.   To improve the lesson’s structure, it may be helpful to incorporate shorter and more frequent exercise intervals, allowing the body to maintain a more consistent level of physical activity.

This can help to reduce fluctuations in pulse rate and other physiological responses, making the exercise stress more manageable for the body. Additionally, it may be helpful to incorporate warm-up and cool-down periods to prepare the body for exercise and allow for recovery afterward.


82. A patient is 16 y.o. In the last year his behaviour has gradually changed: he secluded himself, was not interested in communication with friends, in learning. He became indifferent towards relatives, motivelessly rude, was speaking or laughing to himself. He answers the questions formally correctly, laconically. Considers himself to be absolutely healthy but a little tired, says, he’s thinking about writing a book “Projection of humanity on the plane of Universe”. He always has a copybook with which is full of a great many of the same daggers. What is the most probable diagnosis?

A. Schizophrenia

B. Depressive disorder

C. Schizoid personality disorder

D. Autistic personality disorder

E. Pick’s disease

Answer: Schizophrenia

Explanation

The patient’s behavior is consistent with the positive and negative symptoms of schizophrenia. The gradual change in behavior, seclusion, loss of interest in communication and learning, indifference towards relatives, and motiveless rudeness are all negative symptoms of schizophrenia.

The patient’s speaking or laughing to himself, formal and laconic responses to questions, and belief that he is healthy but a little tired are all positive symptoms of schizophrenia.   The patient’s interest in writing a book on a grandiose theme, “Projection of humanity on the plane of Universe,” is another characteristic feature of schizophrenia.

The presence of a copybook with a great many of the same daggers may also indicate a thought disorder, which is a common feature of schizophrenia.   Depressive disorder and schizoid personality disorder may present with some similar symptoms, such as social withdrawal and loss of interest in activities, but they do not typically include the positive symptoms of schizophrenia, such as delusions and hallucinations.  

Autistic personality disorder and Pick’s disease are also unlikely diagnoses, as they typically present with different symptoms and do not include the characteristic positive and negative symptoms of schizophrenia.   Therefore, based on the patient’s symptoms and behavior, the most probable diagnosis is schizophrenia.

Early diagnosis and treatment of schizophrenia can improve outcomes and quality of life for patients, so it is important to seek professional evaluation and support if this condition is suspected.


83. A worker was temporarily off work because of illness for 16 days, was under out-patient treatment. The doctor in charge issued a sick-list first for 5 days, then prolonged it for 10 days. Who can further prolong the sick-list of this patient?

A. The doctor in charge of the case together with the head of department

B. Working ability expertise committee

C. The doctor in charge of the case with the permission of the head of department

D. Deputy head physician on the working ability expertise

E. The head of department


Answer: The doctor in charge of the case together with the head of department

Explanation

The person(s) responsible for issuing a medical certificate of incapacity for work may vary depending on the specific policies and regulations of the country or healthcare system in question. However, in general, the most appropriate answer to this question would be A. The doctor in charge of the case together with the head of department.  

In many healthcare systems, the doctor in charge of the case is responsible for assessing the patient’s medical condition and determining if they are fit for work. However, in some cases, the head of department or another senior physician may need to be consulted to provide additional expertise or guidance.  

In cases where there is a dispute or disagreement regarding the patient’s fitness for work, a working ability expertise committee may be convened to review the case and provide a final determination.   The deputy head physician on the working ability expertise and the head of department may also be involved in the process, depending on the specific policies and regulations of the healthcare system.  

Therefore, the most appropriate answer to this question would be A. The doctor in charge of the case together with the head of department, as they are typically responsible for assessing the patient’s medical condition and determining their fitness for work.


84. The results of a separate diagnostic curettage of the mucous of the uterus’ cervix and body made up in connection with bleeding in a postmenopausal period: the scrape of the mucous of the cervical canal revealed no pathology, in endometrium – the highly differentiated adenocarcinoma was found. Metastases are not found. What method of treatment is the most correct?

A. Surgical treatment and hormonotherapy

B. Surgical treatment + chemotherapy

C. Surgical treatment and radial therapy

D. Radial therapy

E. –


Answer: Surgical treatment and hormonotherapy

Explanation

The most appropriate treatment for breast cancer depends on several factors, including the stage of the cancer, the patient’s age and overall health, and the characteristics of the tumor, such as hormone receptor status. Based on the limited information provided in this question, the most appropriate answer would be A. Surgical treatment and hormonotherapy.  

Surgical treatment, such as lumpectomy or mastectomy, is typically the first-line treatment for breast cancer. The goal of surgery is to remove the cancerous tissue and determine the extent of the disease. Depending on the size and location of the tumor, lymph nodes in the area may also be removed or sampled to determine if the cancer has spread.  

Hormonotherapy, also known as endocrine therapy, is a type of treatment that blocks the effects of hormones, such as estrogen or progesterone, on cancer cells. It is typically used for breast cancers that are hormone receptor-positive, meaning they have receptors for estrogen or progesterone on their surface.

Hormonotherapy can be given either before or after surgery, and can help to reduce the risk of recurrence and improve outcomes. Chemotherapy and radial therapy may also be used in the treatment of breast cancer, depending on the specific characteristics of the tumor and the patient’s individual situation.

Chemotherapy is typically used for more aggressive or advanced tumors, while radial therapy may be used to target any remaining cancer cells after surgery.   However, without additional information regarding the stage and characteristics of the breast cancer in this case, it is difficult to definitively determine the most appropriate treatment plan.

It is important for patients with breast cancer to work closely with their healthcare team to develop an individualized treatment plan that takes into account their unique medical history, preferences, and goals.


85. A 27 y.o. woman complains of having the disoders of menstrual function for 3 months, irregular pains in abdomen. On bimanual examination: in the dextral appendage range of uterus there is an elastic spherical formation, painless, 7 cm in diameter. USI: in the right ovary – a fluid formation, 4 cm in diameter, unicameral, smooth. What method of treatment is the most preferable?

A. Prescription of an estrogen-gestogen complex for 3 months with repeated examination

B. Operative treatment

C. Dispensary observation of the patient

D. Anti-inflammatory therapy

E. Chemotherapeutic treatment

Answer: Prescription of an estrogen-gestogen complex for 3 months with repeated examination

Explanation

The most appropriate method of treatment for a 27-year-old woman with irregular menstrual function and a 7 cm elastic spherical formation in the dextral appendage range of uterus and a 4 cm unicameral fluid formation in the right ovary would depend on the diagnosis.

However, based on the information provided, the most preferable method of treatment would be A. Prescription of an estrogen-gestogen complex for 3 months with repeated examination.   The presence of an elastic spherical formation in the dextral appendage range of uterus and a unicameral fluid formation in the right ovary suggests the possibility of an ovarian cyst or a uterine fibroid.

These diagnoses are relatively common in young women and are usually benign.   Prescribing an estrogen-gestogen complex for 3 months is a common approach to managing ovarian cysts or uterine fibroids that are not causing severe symptoms or complications. The estrogen-gestogen complex can help to regulate menstrual function and reduce the size of the cyst or fibroid.

However, it is important to closely monitor the patient with repeated examinations to ensure that the cyst or fibroid is not growing or causing any complications. If the cyst or fibroid continues to grow or causes severe symptoms, such as pain or heavy bleeding, operative treatment may be necessary.  

Dispensary observation and anti-inflammatory therapy may also be appropriate in some cases, depending on the specific diagnosis and severity of the symptoms. Chemotherapeutic treatment is typically not indicated for benign ovarian cysts or uterine fibroids.  

Therefore, based on the information provided, the most preferable method of treatment would be A. Prescription of an estrogen-gestogen complex for 3 months with repeated examination. However, it is important to follow up closely with the patient to ensure that the cyst or fibroid is not causing any complications and to adjust the treatment plan as necessary.


86. A 40 y.o. patient complains of yellowi-sh discharges from the vagina. Bimanual examination: no pathological changes. The smear contains Trichomonas vaginalis and blended flora. Colposcopy: two hazy fields on the front labium, with a negative Iodum test. Your tactics:

A. Treatment of specific colpitis and with the subsequent biopsy

B. Diathermocoagulation of the cervix of the uterus

C. Specific treatment of Trichomonas colpitis

D. Cervix ectomy

E. Cryolysis of cervix of the uterus


Answer: Treatment of specific colpitis and with the subsequent biopsy

Explanation

The most appropriate tactic for a 40-year-old patient with yellowish discharges from the vagina, Trichomonas vaginalis and blended flora on smear, and two hazy fields on the front labium with a negative iodine test on colposcopy would be to treat the specific colpitis and perform a subsequent biopsy.

Therefore, the correct answer is A.   The presence of Trichomonas vaginalis and blended flora on smear indicates the possibility of an infection, which can cause inflammation of the cervix and vagina. The two hazy fields on the front labium observed on colposcopy may also indicate inflammation or infection.

However, a negative iodine test suggests that the cells are not abnormal or precancerous.   The most appropriate initial management of this patient would be to treat the specific colpitis with antibiotics or antifungal medications, depending on the specific diagnosis. The subsequent biopsy can help to determine if there are any abnormal or precancerous cells present, and can guide further management if necessary.  

Diathermocoagulation, cryolysis, and cervixectomy are not appropriate treatments for this patient, as there is no indication of abnormal or precancerous cells on colposcopy.   Therefore, the most appropriate tactic for a 40-year-old patient with yellowish discharges from the vagina, Trichomonas vaginalis and blended flora on smear, and two hazy fields on the front labium with a negative iodine test on colposcopy would be to treat the specific colpitis and perform a subsequent biopsy.


87. A full-term new-born suffered from ante- and intranatal hypoxia, was born in asphyxia (Apgar score 2-5 points). After birth baby’s excitation is progressing, occurs vomiting, nystagmus, spasms, squint, spontaneous Babinski and Moro’s reflexes. What is the most probable location of the intracranial hemorrhage in this case?

A. Subarachnoid hemorrhages

B. Small hemorrhages in brain tissue

C. Subdural hemorrhages

D. Periventricular hemorrhages

E. Hemorrhages in ventricles of brain


Answer: Subarachnoid hemorrhages

Explanation

The most probable location of the intracranial hemorrhage in a full-term newborn who suffered from ante- and intranatal hypoxia, was born in asphyxia (Apgar score 2-5 points), and is presenting with vomiting, nystagmus, spasms, squint, spontaneous Babinski and Moro’s reflexes is subarachnoid hemorrhages.

Therefore, the correct answer is A.   Subarachnoid hemorrhages are a type of intracranial hemorrhage that can occur in neonates who have suffered hypoxia or birth asphyxia. The hemorrhage is located between the arachnoid and pia mater layers of the meninges, which surround the brain and spinal cord.  

The symptoms described in the scenario, such as vomiting, nystagmus, spasms, squint, spontaneous Babinski and Moro’s reflexes, are consistent with the neurological signs of subarachnoid hemorrhages in neonates.  

Small hemorrhages in brain tissue, subdural hemorrhages, periventricular hemorrhages, and hemorrhages in ventricles of brain are other types of intracranial hemorrhages that can occur in neonates, but they typically present with different symptoms and neurological signs.  

Therefore, based on the information provided, the most probable location of the intracranial hemorrhage in this case is subarachnoid hemorrhages. Early diagnosis and management of subarachnoid hemorrhages in neonates is important to prevent further neurological damage and improve outcomes.


88. A 37 y.o. primigravida woman has been having labor activity for 10 hours. Labor pains last for 20-25 seconds every 6-7 minutes. The fetus lies in longitude, presentation is cephalic, head is pressed upon the entrance to the small pelvis. Vaginal examination results: cervix of uterus is up to 1 cm long, lets 2 transverse fingers in. Fetal bladder is absent. What is the most probable diagnosis?

A. Primary uterine inertia

B. Secondary uterine inertia

C. Normal labor activity

D. Discoordinated labor activity

E. Pathological preliminary period


Answer: Primary uterine inertia

Explanation

The most probable diagnosis for a 37-year-old primigravida woman who has been having labor activity for 10 hours, with labor pains lasting for 20-25 seconds every 6-7 minutes, a fetus lying in longitude, presentation is cephalic, head is pressed upon the entrance to the small pelvis, a cervix of uterus up to 1 cm long, letting 2 transverse fingers in, and fetal bladder absent is primary uterine inertia.

Therefore, the correct answer is A.   Primary uterine inertia is a type of dysfunctional labor in which the uterus fails to contract effectively to dilate and efface the cervix. It is more common in first-time mothers and may be caused by a variety of factors, such as hormonal imbalances, fetal malposition, or psychological stress.  

The patient’s symptoms, such as the duration and frequency of contractions, the length of the cervix, and the absence of the fetal bladder, suggest that labor is not progressing as expected. This may be due to primary uterine inertia, which can cause prolonged labor and increase the risk of complications for both the mother and the baby.  

Secondary uterine inertia, on the other hand, occurs when contractions have been effective but then become weak or stop altogether. This is not consistent with the patient’s symptoms, as she has not yet reached full cervical dilation.   Normal labor activity, discoordinated labor activity, and pathological preliminary period are not consistent with the patient’s symptoms and history.  

Therefore, based on the information provided, the most probable diagnosis for a 37-year-old primigravida woman who has been having labor activity for 10 hours, with labor pains lasting for 20-25 seconds every 6-7 minutes, a fetus lying in longitude, presentation is cephalic, head is pressed upon the entrance to the small pelvis, a cervix of uterus up to 1 cm long, letting 2 transverse fingers in, and fetal bladder absent is primary uterine inertia.

89. A patient at a doctor complains of temperature rise up to 38, 20, edema in the region of his upper lip. Objectively: upper lip is evidently edematic, in the middle of edema there is a cone-shaped swelling. Skin and mucous membrane over it are dark-red. Diagnosis: labial furuncle. A surgeon cut the furuncle, treated the wound with hydrogen peroxide solution and applied a bandage with hypertensive solution. What therapeutic regimen should be recommended?

A. In-patient treatment with common regimen

B. First out-patient treatment, then outpatient treatment

C. Out-patient treatment

D. In-patient treatment with bed rest

E. –


Answer:  In-patient treatment with common regimen

Explanation

The most appropriate therapeutic regimen for a patient with a labial furuncle that has been incised and treated with hydrogen peroxide solution and a hypertonic bandage is in-patient treatment with a common regimen. Therefore, the correct answer is A.   A labial furuncle is a localized infection of a hair follicle on the upper lip, which can cause pain, swelling, redness, and fever.

Treatment typically involves incision and drainage of the furuncle, followed by local wound care and antibiotics if necessary.   In this case, the surgeon has already incised the furuncle and treated the wound with hydrogen peroxide solution and a hypertonic bandage. However, the patient still has a fever and edema in the region of the upper lip, which may indicate that the infection has spread beyond the initial site of the furuncle.  

In-patient treatment with a common regimen, which includes antibiotics, pain management, and close monitoring of the patient’s condition, is recommended to ensure that the infection is properly treated and to prevent complications. The patient may also require additional wound care and follow-up appointments with the surgeon.  

Out-patient treatment may be appropriate for some patients with uncomplicated furuncles that have been successfully incised and drained. However, in this case, the patient’s symptoms and history suggest a more severe infection that requires in-patient treatment.  

Therefore, based on the information provided, the most appropriate therapeutic regimen for a patient with a labial furuncle that has been incised and treated with hydrogen peroxide solution and a hypertonic bandage is in-patient treatment with a common regimen.


90. A 15 y.o. boy was twice attacked by bees, as a result he had severe anaphylactic shock. What is the most effective prophylaxis method?

A. Desensibilisation by means of bee venom extract

B. Prescription of corticosteroids for summer

C. Long-term prophylactic treatment with antihistamines

D. Limitation of outside staying during summer months

E. Protective clothing


Answer: Desensibilisation by means of bee venom extract

Explanation

Desensitization (also known as immunotherapy) is the only effective method of preventing anaphylactic reactions to bee stings. During desensitization treatment, the patient is gradually exposed to increasing amounts of bee venom extract over a period of several months, until they develop tolerance to the venom and no longer experience severe allergic reactions.  

Corticosteroids and antihistamines can be used to treat acute allergic reactions to bee stings, but they are not effective as long-term prophylaxis. Limiting outside staying during summer months and wearing protective clothing can reduce the risk of bee stings, but they are not reliable methods of preventing anaphylactic reactions in individuals who are already sensitized to bee venom.  

Overall, desensitization is the most effective prophylaxis method for individuals with a history of severe allergic reactions to bee stings.


91. A healthy 75 y.o. woman who leads a moderately active way of life went through a preventive examination that revealed serum concentration of common cholesterol at the rate of 5,1 mmol/L and HDL (high-density lipoproteins) cholesterol at the rate of 70 mg/dl. ECG reveals no pathology. What dietary recommendation is the most adequate?

A. Any dietary changes are necessary

B. Decrease of cholesterol consumption

C. Decrease of saturated fats consumption

D. Decrease of carbohydrates consumption

E. Increase of cellulose consumption


Answer: Any dietary changes are necessary

Explanation

While the serum concentration of common cholesterol in this woman is within the normal range, the HDL cholesterol level is high, which is a good thing. However, it is important to note that high cholesterol levels are a risk factor for cardiovascular disease, and at 75 years old, this woman may be at increased risk for developing cardiovascular disease.  

Therefore, it is advisable to make some dietary changes to reduce the risk of cardiovascular disease. This may include decreasing the consumption of saturated fats (found in animal products such as meat and dairy), increasing the consumption of fiber-rich foods (such as fruits, vegetables, and whole grains), and limiting the intake of processed and high-calorie foods.  

It is not necessary to specifically decrease cholesterol consumption, as the body produces cholesterol on its own and dietary cholesterol intake has a limited impact on blood cholesterol levels. Additionally, there is no need to decrease carbohydrate consumption, as carbohydrates are an important source of energy for the body.  

In summary, while this woman’s cholesterol levels may not be cause for immediate concern, making some dietary changes can help reduce the risk of cardiovascular disease in the long term.


92. In treatment and prevention establishments, regardless of their organisational and proprietary form, the rights of patients should be observed. Which of these rights are the most significant?

A. The right to the protection of the patient’s interests

B. The right to the free choice

C. The right to the information

D. The right to be heard

E. The right to the protection from incompetence


Answer: The right to the protection of the patient’s interests

Explanation

All of the listed rights are important for patients in treatment and prevention establishments, but the most significant right is A, the right to the protection of the patient’s interests.   This right encompasses a variety of important aspects, including ensuring that the patient receives appropriate and effective medical care, protecting the patient’s privacy and confidentiality, and safeguarding the patient’s physical and emotional well-being.

In other words, the healthcare provider must act in the best interests of the patient and prioritize their needs above other considerations.  

The right to free choice is also an important right, as patients should be able to choose their healthcare provider and participate in decisions about their medical care. The right to information is another important right, as patients have the right to be informed about their condition, treatment options, and potential risks and benefits.

The right to be heard is also crucial, as patients should be able to express their concerns and preferences and have them taken into account by their healthcare provider. Finally, the right to protection from incompetence is important to ensure that patients receive safe and effective medical care from qualified professionals.  

Overall, all of these patient rights are important and must be upheld in treatment and prevention establishments to ensure that patients receive the highest quality of care. However, the right to the protection of the patient’s interests is the most significant, as it encompasses many other important aspects of patient care.


93. A patient has complained of great weakness for 6 years. He fell seriously ill, the illness is accompanied by body temperature rise, indisposition, pain in joints and along the legs muscles. Objectively: violet-bluish erythema around eyes and over knee joints. HR- 120/min, heart sounds are weak. Blood count: leukocytes – 12 ∗ 109/L, ESR- 40 mm/h. What is the most probable diagnosis?

A. Dermatomyositis

B. Systemic lupus erythematosus

C. Rheumathoid arthritis

D. Atopic dermatitis

E. Reactive polyarthritis


Answer: Dermatomyositis

Explanation

Dermatomyositis is a rare autoimmune disease that affects the muscles and skin. The classic clinical features include symmetric proximal muscle weakness, along with a characteristic rash that appears on the face, neck, shoulders, and upper chest. The rash is violet-bluish in color and is known as a heliotrope rash.

The erythema around the eyes and over the knee joints described in this case are consistent with this rash.   Other common symptoms of dermatomyositis include fatigue, fever, joint pain, and muscle pain. The elevated heart rate and weak heart sounds described in this case may be related to cardiac involvement, which can occur in up to 30% of patients with dermatomyositis.  

The elevated leukocyte count and ESR indicate the presence of inflammation, which is a hallmark of autoimmune diseases such as dermatomyositis. Systemic lupus erythematosus (B) can also present with similar symptoms, but typically does not cause muscle weakness as a primary symptom.

Rheumatoid arthritis (C) can cause joint pain and stiffness, but typically does not cause muscle weakness and does not present with a characteristic rash. Atopic dermatitis (D) is a chronic skin condition characterized by itching and rash, but does not cause muscle weakness or joint pain.

Reactive polyarthritis (E) is a type of arthritis that occurs as a reaction to an infection, but does not typically cause a rash or muscle weakness.   Therefore, based on the symptoms and objective findings described, the most probable diagnosis is dermatomyositis.


94. A military unit stopped for 3-day’s rest in inhabited locality after a long march. The sanitary-epidemiological reconnaissance found several water sources. It is necessary to choose the source complying with the hygienic standards for drinking water in the field

A. Artesian well water

B. Spring water

C. River water

D. Rain water

E. Water from melted snow


Answer: Artesian well water

Explanation

Artesian well water is typically the safest and most reliable source of drinking water in the field, as it is usually free from contamination and meets hygienic standards for drinking water. Artesian wells are usually deep underground and are accessed through a borehole, which helps to protect the water from surface contamination.  

Spring water (B) may also be a good source of drinking water, but it can be more difficult to ensure that the water is free from contamination. River water (C) is generally not a safe source of drinking water, as it is often contaminated with bacteria, viruses, and other pathogens.

Rainwater (D) and water from melted snow (E) may be safe to drink if collected and stored properly, but they can also be contaminated with pollutants and microorganisms.  

In general, it is important to ensure that any source of drinking water in the field is properly treated and tested to ensure that it meets hygienic standards for drinking water. If artesian well water is not available, other sources of water should be evaluated carefully to ensure that they are safe and free from contamination.


95. A district doctor was commisioned with a task to work out a plan of treatment-and-prophylaxis actions for the population of his district. What actions of secendary prophylaxis must he include into this plan?

A. Prevention of disease complications

B. Disease prevention

C. Elimination of disease causes

D. Improvement of population’s living conditions

E. Rehabilitation actions


Answer: Prevention of disease complications

Explanation

The actions of secondary prophylaxis that a district doctor should include in a plan of treatment-and-prophylaxis actions for the population of his district are A, prevention of disease complications.   Secondary prophylaxis refers to actions taken to prevent the recurrence or exacerbation of a disease in individuals who have already been diagnosed with it.

One important aspect of secondary prophylaxis is the prevention of disease complications, which can occur in individuals who have already been diagnosed with a disease and may be at increased risk for developing complications.  

For example, in patients with hypertension, secondary prophylaxis may include regular blood pressure monitoring, medication management, and lifestyle modifications to prevent complications such as heart attack, stroke, and kidney damage. In patients with diabetes, secondary prophylaxis may include regular blood glucose monitoring, medication management, and lifestyle modifications to prevent complications such as nerve damage, foot ulcers, and eye damage.  

Other actions listed, such as disease prevention (B), elimination of disease causes (C), improvement of population’s living conditions (D), and rehabilitation actions (E) are important for primary or tertiary prevention, respectively. Disease prevention involves actions taken to prevent the onset of a disease in individuals who are at risk for developing it.

Elimination of disease causes involves actions taken to remove or reduce the factors that contribute to the development of a disease. Improvement of population’s living conditions involves actions taken to improve the overall health and well-being of the population.

Rehabilitation actions involve actions taken to help individuals recover from a disease or injury and regain their functional abilities. Therefore, the most appropriate action of secondary prophylaxis that a district doctor should include in a plan of treatment-and-prophylaxis actions for the population of his district is prevention of disease complications.


96. A 43 y.o. patient complains of formation and pain in the right mammary gland, rise of temperature up to 37, 20C during the last 3 months. Condition worsens before the menstruation. On examination: edema of the right breast, hyperemia, retracted nipple. Unclear painful infiltration is palpated in the lower quadrants. What is the most probable diagnosis?

A. Cancer of the right mammary gland

B. Right-side acute mastitis

C. Right-side chronic mastitis

D. Premenstrual syndrome

E. Tuberculosis of the right mammary gland


Answer: Cancer of the right mammary gland   

Explanation

The presence of a painful lump in the breast, along with edema, hyperemia, and retracted nipple, are all signs that suggest a malignant (cancerous) process. In addition, the fact that the patient’s symptoms worsen before menstruation is not typical of a benign breast condition such as fibrocystic changes or premenstrual syndrome.  

It is important to note that not all breast lumps are cancerous, and further diagnostic testing such as mammography and biopsy are needed to confirm the diagnosis. However, given the patient’s age and the presence of concerning symptoms and examination findings, cancer of the right mammary gland is the most likely diagnosis.  

Acute mastitis (B) is an infection of the breast tissue that typically presents with redness, warmth, and tenderness. Chronic mastitis (C) is a non-infectious inflammation of the breast tissue that can also cause breast pain and swelling, but typically does not present with a lump or retracted nipple.

Premenstrual syndrome (D) is a collection of symptoms that occur in the days before menstruation and typically involve mood changes, bloating, and breast tenderness, but not the formation of a breast lump. Tuberculosis of the mammary gland (E) is a rare condition that can cause breast lumps and other symptoms, but is typically accompanied by systemic symptoms such as fever and weight loss.

97. A 20 y.o. patient was admitted to the hospital with complaints of having skin and sclera icteritiousness, dark urine, single vomiting, appetite loss, body temperature rise up to 380 for 2 days. Three weeks ago he went in for fishing and shared his dishes with friends. Objectively: the patient is flabby, t 0- 36, 80, skin and scleras are icteritious, liver sticks from under the costal margin by 3 cm, it is sensitive; spleen isn’t palpable. Urine is dark, stool is partly acholic. What is the most probable diagnosis?

A. Virus A hepatitis

B. Leptospirosis

C. Infectious mononucleosis

D. Hemolytic anemia

E. Intestinal yersiniosis


Answer: Virus A hepatitis

Explanation

Viral hepatitis A is a highly contagious viral infection that is transmitted through the fecal-oral route, often through contaminated food or water. The incubation period for hepatitis A is typically 2-6 weeks, which is consistent with the patient’s history of sharing dishes with friends three weeks ago. The onset of symptoms is typically abrupt and includes fever, nausea, vomiting, loss of appetite, and jaundice.   T

he presence of icteric sclera and dark urine, along with the sensitivity of the liver and absence of palpable spleen, are all consistent with hepatitis A. The elevation of liver enzymes in blood tests would confirm the diagnosis. The acholic stool indicates the lack of bilirubin in the stool, which is typical for obstructive jaundice.  

Leptospirosis (B) is a bacterial infection that is transmitted through contact with contaminated water or soil, usually in tropical or subtropical regions. The symptoms include fever, headache, muscle pain, and vomiting, but usually do not include jaundice or liver enlargement.

Infectious mononucleosis (C) is a viral infection caused by the Epstein-Barr virus, and typically presents with fever, sore throat, swollen lymph nodes, and fatigue, but does not typically cause jaundice or liver enlargement. Hemolytic anemia (D) is a condition in which the body destroys red blood cells, leading to anemia and jaundice, but usually does not cause liver enlargement.

Intestinal yersiniosis (E) is a bacterial infection caused by Yersinia enterocolitica, and typically presents with diarrhea, abdominal pain, and fever, but does not typically cause jaundice or liver enlargement.   Therefore, based on the symptoms and examination findings described, the most probable diagnosis is virus A hepatitis.


98. A full-term newborn child has a diagnosis Rh-factor hemolytic disease of newborn. Bilirubin rate is critical. The child’s blood group is В(III), his mother’s blood group – А(II). The child has indication for hemotransfusion. What donor blood must be chosen?

A. Blood group В(III), Rh (-)

B. Blood group А(II), Rh (-)

C. Blood group B(III), Rh (+)

D. Blood group А(II), Rh (+)

E. Blood group О(I), Rh (-)


Answer: Blood group В(III), Rh (-)

 Explanation

In the case of Rh-factor hemolytic disease of the newborn, the baby’s red blood cells are destroyed due to an incompatibility between the mother’s and the baby’s blood type and Rh factor.

In this case, the baby has blood group B(III) and the mother has blood group A(II), which means that the baby inherited the B antigen from the father and the Rh factor from either parent.   To avoid further hemolysis, it is necessary to transfuse the baby’s blood with compatible blood.

In general, for a baby with Rh-factor hemolytic disease, it is important to choose a donor blood that is negative for the Rh factor and does not have the antigen that caused the hemolysis. Therefore, the most appropriate donor blood for this baby would be A or AB blood group, Rh negative. However, since the baby’s blood type is B, it is preferable to choose a donor blood of the same blood group to avoid additional immune reactions.  

Therefore, the most appropriate donor blood for this baby would be blood group B(III), Rh negative, which is the same blood group as the baby’s and does not contain the antigen that caused the Rh-factor hemolytic disease. Option A is the correct answer.


99. At year-end hospital administration has obtained the following data: annual number of treated patients and average annual number of beds used for patient’s treatment. What index of hospital work can be calculated based upon this data?

A. Bed turnover

B. Bed resources of the hospital

C. Average annual bed occupancy

D. Average duration of patients presence in the hospital

E. Average bed idle time


Answer: Bed turnover

Explanation

Bed turnover is a measure of the rate at which hospital beds are used, and is calculated by dividing the number of admissions (or discharges) by the average number of beds. In this case, the annual number of treated patients and the average annual number of beds used for patient’s treatment are both available, so it is possible to calculate the bed turnover.  

Bed resources of the hospital (B) refers to the total number of beds in the hospital, rather than the number of beds used for patient treatment. Average annual bed occupancy (C) is a measure of the percentage of beds that are occupied over a given period of time, and is calculated by dividing the total number of occupied bed days by the total number of available bed days.

Average duration of patients presence in the hospital (D) is a measure of the length of stay for patients in the hospital, and is calculated by dividing the total number of patient days by the total number of discharges.

Average bed idle time (E) is a measure of the amount of time that beds are unoccupied, and is calculated by subtracting the total number of occupied bed days from the total number of available bed days.   Therefore, the most appropriate index of hospital work that can be calculated based on the data provided is bed turnover.


100. A 52 y.o. patient fell from 3 m height on the flat ground with the right lumbar area. He complains of pain in this area. There is microhematuria in the urea. Excretory urography revealed that kidney’s functioning is satisfactory. What is the most probable diagnosis?

A. Kidney’s contusion

B. Subcapsular kidney’s rupture

C. Multiple kidney’s ruptures

D. Paranephral hematoma

E. Kidney’s abruption


Answer: Kidney’s contusion

Explanation

A kidney contusion is a bruise or injury to the kidney that occurs as a result of a direct blow or trauma to the lower back or abdomen. The patient’s history of falling from a height onto the right lumbar area is consistent with this type of injury.

Microhematuria, or the presence of blood in the urine, is a common finding in patients with kidney contusions.   Excretory urography revealed that kidney’s functioning is satisfactory, which suggests that the kidney is not significantly damaged or ruptured.

Subcapsular kidney’s rupture (B) or multiple kidney’s ruptures (C) would likely result in a more severe injury and impaired kidney function. Paranephral hematoma (D) is a collection of blood near the kidney, usually caused by trauma, but it does not involve injury to the kidney itself.

Kidney’s abruption (E) is a rare condition in which the kidney is completely separated from its blood supply, usually as a result of trauma, and is typically accompanied by severe pain and bleeding.   Therefore, based on the clinical presentation and examination findings described, the most probable diagnosis is kidney’s contusion.
Join the conversation
0% Complete